PiR2
Gostaria de reagir a esta mensagem? Crie uma conta em poucos cliques ou inicie sessão para continuar.

Indução - Putnam 2009

2 participantes

Ir para baixo

Indução - Putnam 2009 Empty Indução - Putnam 2009

Mensagem por gabriel_balbao Ter 08 Mar 2022, 23:07

(Putnam 2009) Para n inteiro positivo, mostre que

[latex]\sum_{k=1}^{n}\sqrt{k} < \frac{4n + 3}{6}.\sqrt{n}[/latex]


Não queria a resolução diretamente. Se puderem deixar umas dicas/ideias antes da resolução (ou a resolução como spoiler), ajudaria bastante. 
gabriel_balbao
gabriel_balbao
Padawan
Padawan

Mensagens : 92
Data de inscrição : 03/02/2021
Idade : 20
Localização : Ribeirão Preto

Ir para o topo Ir para baixo

Indução - Putnam 2009 Empty Re: Indução - Putnam 2009

Mensagem por joaoZacharias Dom 20 Mar 2022, 14:20

Olá Gabriel;

Defina-se [latex]S_n = \sum_{k=1}^{n}\sqrt{k} [/latex]

Então [latex] S_{n+1} = S_n + \sqrt{n+1}[/latex]

Suponha que para algum n tenha-se [latex] S_{n} < \frac{4n + 3}{6}\sqrt n[/latex]. Nessas condições:

[latex]\scriptsize{ S_{n+1} - \sqrt{n+1} = S_n \text{ }, \text{ } \text{ } S_{n} < \frac{4n + 3}{6}\sqrt n \implies (S_{n+1} - \sqrt{n+1}) < \frac{4n + 3}{6}\sqrt{n} \implies S_{n+1} < (\sqrt{n+1} + \frac{4n + 3}{6}\sqrt{n})}[/latex]

Deseja-se provar que:

[latex]S_{n+1} < (\frac{(4(n+1) +3)}{6}\cdot \sqrt{n+1})[/latex]

Hipótese inicial:
Spoiler:

Verificação da hipótese:
Spoiler:

Bons estudos Very Happy
joaoZacharias
joaoZacharias
Recebeu o sabre de luz
Recebeu o sabre de luz

Mensagens : 134
Data de inscrição : 18/03/2020
Localização : Campinas - SP, BR

qedpetrich gosta desta mensagem

Ir para o topo Ir para baixo

Ir para o topo

- Tópicos semelhantes

 
Permissões neste sub-fórum
Não podes responder a tópicos